求解这道大学物理题,要详细过程

如题

(1)P点场强大小 E = 2 q / (4π ε0 (L^2+L^2)) * cos45° = 1.91 V/m
方向向左
(2)P点电势 V = 0
附注:我的回答常常被“百度知道”判定为违反“回答规范”,但是我一直不知道哪里违规,也不知道对此问题的回答是否违规。
温馨提示:内容为网友见解,仅供参考
无其他回答
相似回答